Sciencemadness Discussion Board

Solar energy a pipe dream?

smuv - 16-7-2011 at 21:34

I was discussing solar energy with someone, and it got me thinking I should run the numbers to try to figure out assuming (unrealistically) 100% efficient capture, how much of the earth needs to be covered with solar panels to fulfill global energy consumption. The number I got out, was 17,000 sq miles. Although this works out to be about half of Portugal in area, (or about 0.009% of the earth) is it feasable that we could even build solar panels/concentrators on this order of magnitude?

I know a lot of variables are being left out (such as variance of solar energy by location and efficiency of panels) but still, this is a generous ballpark figure to analyze feasibility.

What are your thoughts? Am I a pessimist?

How I calculated
-Total energy from sun that strikes face of earth each year: 5.5×10^24 J (wikipedia)
-Total world yearly energy consumption (2008): 4.74x10^20 J (wikipedia)
-Surface area of earth: 57510000 sq mi

Formula: Energy consumption/Suns energy * Area of earth

Idiot check: A book (Industrial Organic chemistry) quoted 0.005% of the earths surface, which is lower than what I calculated but still within an order of magnitude.

AndersHoveland - 16-7-2011 at 23:08

Solar towers utilizing steam turbines will offer a cheaper and more efficient source of electric power than solar panels.

The earth's surface has plenty of deserted areas to build vast arrays of solar panels. The problem is getting the electric current to populated areas. There are significant loses from resistance over long distances. Superconductive electric lines would be a solution, but the type of superconductors that can handle high current require expensive liquid-helium cooling. Solar panels can be built into commercial building rooftops, and this can provide sufficient power for most non-industrial purposes. Most of the electricity used goes to manufacturing. If there is to be any real progress made in the world at converting to solar power, the western governments need to provide strong incentives to China, as much of the products consumed in the west are actually produced in China. Simply forcing factories to use expensive solar power will just drive all the manufacturing away to other countries that use cheap power from inefficient polluting sources. It is almost completely futile for the governments to think that getting people to use energy-efficient compact fluorescent lighting in their home will make any difference (I also really hate those spiral lights). I do not know why they use fluorescent lighting in schools, it makes it harder for many of the students to concentrate, and it is much less pleasant than the soft glow of an incandescent bulb. There is always LED, but that apparently is "too expensive", even though LED lights last an extremely long time (typically over 20 years) before they burn out.

The main problem is that using solar power is much more expensive. At least five times as expensive as coal-derived power. Taxing carbon dioxide emissions, and putting tariffs on Chinese goods based on pollution, would be the best way to shift to using renewable energy sources. The best way to ensure the use of more renewable resources is to provide strong incentives for large businesses to use less energy, not just to simply use more "efficient" devices.

I think harnessing the energy in ocean waves would be the best option. Waves have a higher energy density, so it would be easier to harness their power. There is also plenty of available coast in the more northern regions in which the weather is more cloudy, making solar panels less effective.

Mixell - 16-7-2011 at 23:15

Actually, liquid helium cooled superconductors are out-dated, there are already liquid nitrogen cooled ones, although mass producing them is not affordable at the moment (as with the helium cooled ones).

Polverone - 17-7-2011 at 01:32

Solar energy is not a pipe dream, though it is not suited for all places and even in suitable places it should not be expected to provide 100% of energy.

The first point to consider is that not all joules are created equal. Naive or disingenuous solar critics may cite the enormous potential energy embedded in oil, and the tremendous difficulty of replacing all those joules with solar electricity. The challenge is daunting, but not as bad as a simple joule-for-joule comparison would suggest. For example, under favorable conditions an efficient small car like the Ford Fiesta can travel 1 kilometer using 0.059 liter (2.05 megajoules) of gasoline. A Nissan Leaf can travel 1 kilometer using 200 watt hours (0.72 megajoules) of electricity. The Leaf makes nearly 3 times better use of its joules because electric motors can turn electricity into motion very efficiently.

Likewise, replacing gas or fuel oil for warming homes and buildings does not require a one-for-one joule replacement by electricity. Heat pumps can use less than 1 joule of energy to add 1 joule to the inside air temperature, at least until the outside air temperature gets very low. Even under very cold conditions ground coupled heat pumps can continue to warm air more efficiently than plain resistive heating.

Heating and cooling consume the lion's share of energy in homes and building, and these functions can be performed with solar-thermal inputs as well as solar-electric. Warming water and air can be done much more cheaply with direct thermal absorption systems than with electrical systems. Even air conditioning and refrigeration (absorption refrigeration systems) can be driven by relatively cheap, low-grade solar heat. Further, new structural designs (or simply insulation upgrades in existing structures) can substantially reduce the energetic inputs required for climate control.

According to a technical report by the National Renewable Energy Laboratory, there is sufficient space on existing roof tops in the USA for PV solar to supply 819 terawatt hours annually; for comparison, the USA's nuclear reactors produced about 800 terawatt hours in 2010, out of about 4000 terawatt hours of total electric generation. The base case scenario in the NREL study assumes only 13.5% solar conversion efficiency. Silicon based modules are already commercially available with greater than 20% conversion efficiency. At 20%, the rooftop potential is 1213 terawatt hours, or about 30% of total current electric generation in the USA. The scale of deployment would be vast, to be sure, but it requires no improvements in PV efficiency, no rare elements or complex multi-junction cells, no additional land consumption, and no new long distance transmission lines.

There's already more than enough land, more than good enough conversion efficiency, and more than enough joules for the taking from the sun*. The greatest problem is the variability of solar generation with time and location, which means that supply is poorly matched to demand much of the time. Matching variable renewable supply and demand probably requires some combination of demand management, extra transmission capacity, and energy storage. This is complicated and expensive. I personally expect that in the sunniest regions PV electricity will be less expensive than coal-fired electricity before the end of the decade, but only if it can be used immediately and locally without storage or long-distance transmission. It is an interesting challenge to consider what processes might be designed or redesigned to profitably use cheap electricity that comes from variable and low-capacity-factor generation.

*Note that wind and conventional hydroelectric power are (indirectly) also forms of solar energy.

bbartlog - 17-7-2011 at 05:38

Quote:
Solar towers utilizing steam turbines will offer a cheaper and more efficient source of electric power than solar panels.


Cheaper? What makes you think so? Plumbing and moving parts for high-temperature working fluids tend to be expensive and have low MTBF. Granted the reflectors are cheaper than a photovoltaic panel, but I don't think you come out ahead.

Polverone - 17-7-2011 at 09:19

Quote: Originally posted by bbartlog  
Quote:
Solar towers utilizing steam turbines will offer a cheaper and more efficient source of electric power than solar panels.


Cheaper? What makes you think so? Plumbing and moving parts for high-temperature working fluids tend to be expensive and have low MTBF. Granted the reflectors are cheaper than a photovoltaic panel, but I don't think you come out ahead.


Solar thermal plants scrap steam for photovoltaic

Some large projects are swapping PV for solar thermal because there are more deployed PV systems as data points for risk modeling, PV costs have declined faster than thermal, and PV siting is more flexible. Another point, not mentioned in the article, is that PV needs less water to operate than thermal based plants.

In my opinion, the biggest downside of going to PV from thermal is the missed opportunity for relatively low cost energy storage. Thermal systems that directly heat molten salts can buffer energy at comparatively low cost just by adding larger insulated storage tanks and more salt mixture, so that system electrical output can be matched to demand much better than a simple PV system. At current low penetration rates of intermittent renewables I suppose there's not much material incentive to take demand-matching into account for project ROI.

Morgan - 17-7-2011 at 10:28

My neighbor a few houses down just got something like this for his roof.
http://www.npr.org/templates/story/story.php?storyId=1279853...


smuv - 17-7-2011 at 10:45

I guess we will see. I would love solar to provide cheap renewable energy, but the scale needed to deploy it makes me skeptical. We are not talking about building a few solar sites here and there, it would have to be a MASSIVE effort just to make a dent. The quoted all rooftops to = power of nuclear, means that if everyone put solar panels on their roof, you only get 8-9% of U.S. ELECTRICITY generation not energy consumption. Which is really just a drop in the bucket. Another thing is, last I checked in the northeast and other regions of the country, over the lifetime of the solar panel you barely get back the energy required to manufacture it.

I guess in some places, solar makes sense, but I think a lot of people look at it as a silver bullet, and it is simply not.

Morgan - 17-7-2011 at 11:30

Solar Stirling motor 3kW infinia
http://www.youtube.com/watch?v=VEpq-WCTOrM

Stirling Dish 500MW solar power plant
http://www.youtube.com/watch?v=OTQ4cFn5sXs

[Edited on 17-7-2011 by Morgan]

Polverone - 17-7-2011 at 13:03

Quote: Originally posted by smuv  
I guess we will see. I would love solar to provide cheap renewable energy, but the scale needed to deploy it makes me skeptical. We are not talking about building a few solar sites here and there, it would have to be a MASSIVE effort just to make a dent. The quoted all rooftops to = power of nuclear, means that if everyone put solar panels on their roof, you only get 8-9% of U.S. ELECTRICITY generation not energy consumption. Which is really just a drop in the bucket. Another thing is, last I checked in the northeast and other regions of the country, over the lifetime of the solar panel you barely get back the energy required to manufacture it.

I guess in some places, solar makes sense, but I think a lot of people look at it as a silver bullet, and it is simply not.


Actually, nuclear supplies about 20% of US electricity consumption, and as I mentioned above rooftop solar could provide 30% if you assume the use of higher-efficiency panels instead of the the rather poky 13.5% in the NREL base case scenario. Also as mentioned above, substituting electricity for other forms of energy generally means that you need fewer total joules to get the same amount of useful work done. For example, even in the base case, rooftop solar provides enough electricity to replace all gasoline used by passenger commute in the US, if electric vehicles were substituted for internal combustion engine vehicles.

I don't think there is a single silver bullet that can assure affordable, reliable energy for the long term without terrible side effects. For my part I favor nuclear energy, renewable electricity, and (while it remains available) natural gas -- basically Anything But Coal. I think that solar can provide a significant part of the energy needed to power a comfortable technologically advanced civilization. If you look at a PV solar resource map of the United States, assuming stationary panels, even Arizona gets less than double the annual solar energy that Maine gets. Solar panels can easily repay their manufacturing energy costs, and more, in any state in the continental US. It is the financial payback that is still difficult.

As recently as 2008 I considered PV solar an expensive joke unless you lived in a very sunny place with no electrical grid. It's still expensive, even compared to other forms of renewable energy, but it's made impressive strides in increasing scale and cutting manufacturing cost in the last few years. There are credible, substantial cost-reduction measures still to play out over the rest of the decade. And unlike wind, wave, or hydro power, solar PV can easily scale down to a single building or home, so it can compete directly against retail electricity costs to the end-user rather than against generation costs to a utility.

Polverone - 17-7-2011 at 13:15

Quote: Originally posted by Morgan  
Solar Stirling motor 3kW infinia
http://www.youtube.com/watch?v=VEpq-WCTOrM

Stirling Dish 500MW solar power plant
http://www.youtube.com/watch?v=OTQ4cFn5sXs

[Edited on 17-7-2011 by Morgan]


Oh, that's interesting. It looks like the company started right in my town and so did the first commercial demonstration project. 24% conversion is very good and with dual-axis tracking they will be able to capture a lot of solar energy. The main downside is that the siting options are more limited compared with PV, since light diffused through cloudy conditions is useless for it.

Morgan - 17-7-2011 at 13:49

Just a razzle dazzle demo/presentation for the general viewing audience - I guess it would be fun to play with.

Jem Melts Rock Using Sunshine
http://www.youtube.com/watch?v=z0_nuvPKIi8

smuv - 17-7-2011 at 14:47

Touche polverone. I'm glad you made those points, makes me feel better about our energy future.

But for sure, that is A LOT of solar panels! But I guess if its cheaper than coal...it will happen.

I like the PV resource map link.

[Edited on 7-17-2011 by smuv]

Endimion17 - 18-7-2011 at 10:23

The economy argument has many flaws. People will buy everything, even if it's very expensive and unnecessary. And people that will use that for their own profit exist. There are quite lots of them, too.

The problem with solar energy is people failing to understand that there are at least three possible energy benefits from it.
First of all, we all use the energy of the Sun, solar or thermal. We grow food using photosynthesis, we heat our homes using infrared rays. Seems like nothing, but it's a lot.
Second thing, there's light to electricity, and the third one is heat, or pure thermal energy.

First one is self explanatory.
Second one is usage of photovoltaic elements, and that's the main problem.
Third is thermal, which can be really, really great for reducing the consumption of electricity per capita, but not everywhere, and not always.

Regarding photovoltaics, in most cases you get a bad Chinese product that was made by heavy industry that sucks ass when it comes to basic filtering and containing the pollution. Greenhouse gasses, heavy metal, hydrogen fluoride emissions, etc.
That product, applied in the temperate geographical zones inhabited by soccer moms and hippies will probably never work long enough to pay itself off - in joules of energy. In dollars it will, of course. As I've said, people will buy everything. Economy is a bitch.

Putting PV in, let's say London, is laughable. It's a terrible economic burden, and somewhere on Earth, tons of coal were turned to toxic smoke and soot just to make a soccer mom think she's "enviromentally conscious".

The amount of pollution that is produced for PV (per joule) greatly exceeds the one from coal energy. Yeah, coal produces lots of pollution, but also gives a lot of energy. PV in temperate zone produces massive pollution, but gives bits and pieces. And it doesn't work at night, so you have to use chemical batteries, which are ridiculously inefficient, require heavy pollution to be produced, and have a short lifespan.

And to think there are actually so called "green activists" that support PV together with electric cars (which are enviromentally viable only in certain cases) is scary, because they're gaining power and influence.


Light-to-electricity has also one problem, from an engineering standpoint, which you forgot to mention. When such "renewable energy sources" reach some 20-30% of the whole net energy production, the system starts to fail because each of these power plants require too much backup.
And guess which one is it? Gas. When cloud obscures the sunlight, there's a sharp drop in the produced energy, and if you want your country/city to have a proper operating machines and appliances, you have to ensure there are no gaps and peaks in voltage and frequency. Something similar happens with windpower, although that source is much more viable and less bullshity. There were some power drops and surges that knocked power gridswhich was caused by such excessive relying on unstable sources.

So if you give 100 kW using sunlight, you need 100 kW of gas waiting to quickly jump in. Double the initial pollution.


PV is great if it's too expensive to lay a powerline to a remote location which has plenty of sunlight and doesn't require lots of energy. And it's great for leveling peak consumption in places in the tropics and close to it.
But trying to use it for the base of the power system in a temperate country is incredibly stupid from an engineering, energy and enviromental standpoint.

Experts in the power industry know it, just as the professors at faculties. Those people often do a facepalm when confronted with hippie BS.

And yeah, I'm not one of those capitalist "let's burn all the fossils and spend everything" kind of guys. I really am pro-enviroment, but I've got a brain, too.

Polverone - 18-7-2011 at 12:45

I think you have it backwards. Energy payback is not hard to get from PV systems, but financial payback is. Financial payback currently requires remote off-grid applications or government incentive schemes like feed-in tariffs. It is unfortunately difficult to evaluate the "real" cost of energy from different sources because all energy producers enjoy various direct and indirect government supports everywhere but Somalia, and in different ways so it's hard to normalize for comparison. Different technologies also aren't held to the same standards in controlling their negative externalities. Coal plants, for example, annually contribute to tens of thousands of premature deaths from smokestack emissions. Over the last 25 years, even including Chernobyl and Fukushima, nuclear power has directly and indirectly killed fewer people than coal power does every year. But most people seem to fear cancer from radiation more than cancer from smoke, so coal gets a free pass.

On the matter of PV energy payback times:

This 2004 NREL study, based on older yet data, estimates about 3.7 years for energy payback using insolation levels equal to the average for the continental United States.

This 2005 study estimates maximum energy payback times of 2.2 years for silicon PV in southern Europe.

IEEE Spectrum: How Free is Solar Energy? Even in Moscow energy payback time for current silicon PV is just over 3 years. If systems last less than 20 years they still have a healthy energy payback ratio. In Phoenix, Arizona the energy payback time is under 2 years.

On the matter of dirty PV production: it is true that producing PV systems requires toxic chemicals and produces negative environmental externalities, particularly when production is located in jurisdictions with weak or poorly enforced environmental regulations. But so does producing the industrial equipment for any energy generation scheme. Gas turbines, diesel engines, and coal plants don't grow on trees either, and over their life cycles they have worse environmental externalities per kilowatt hour of energy produced.

I am one of those crazy people who supports electric and plug-in hybrid vehicles and hopes to see increased PV electrical production. I see them as complementary moves. Electric vehicles can substantially reduce demand for expensive, dwindling, and politically volatile oil, and they can take better advantage of intermittent renewables than most electric loads. If I have an electric vehicle that I use for errands and commuting to work, I only care that its battery is topped up each morning when I go to use it. I don't care when during the day or night it is recharged. The vehicle can charge when renewable sources are peaking/less expensive and defer charging when electricity is less available/more expensive. I agree that the electric grid cannot tolerate a large fraction of intermittent renewables without storage at present, and perhaps not ever, but it's going to take a while for the USA to even approach the 20% limit.

Endimion17 - 18-7-2011 at 14:11

I am very skeptical regarding the graph where m(CO2)/kWh is lower for PV than the one for nuclear power. That seems like a joke. I'm not buying it. Fission is orders of magnitude more efficient, and its carbon footprint should be way smaller than PV's.
PV carbon footprint depends on the place where it's utilized, and even if you put it in Dubai, it shouldn't be close to fission. It's way less than coal, but can't be close to fission.
And I disagree with the energy return. It can't be close to 2-3 years. Few years ago, it was "more than 10 years". Usually close to 15. During the 80's, it was over 20.
AFAIK, it's close to 5 years, if there's enough sunlight through the year. That is the fact which is always missing.

Those things break apart easily, if there's lots of rain. I'd imagine a high quality PV running for a decade or more in an arid area, but in Frankfurt, Paris... 5 years tops, and then water creeps inside.

I am pro-PV, but only where it's viable, complementary and if used by the experts' directive, not money, like it often happens.
That is during the day in places where enough sunlight is plentiful, but limited to the peak thirst for power. PV's can not be the base of power production. The base is coal, hydro (if available) and fission. It is simply impossible to supply a modern city using just, or predominantly PV. It is too weak, too expensive and too unreliable.

I really dislike coal, but that is the only option for a country that doesn't have much hydropotential left, and its citizens are unfortunately furiously opposing fission.
Coal has that nasty drawback that it really causes a whole bunch of seemingly hidden deaths. The stacks send not only chemical poisons, but also radionuclides. Way, way more than any normal operating fission plant. Not many people know that. They usually think that the steam from cooling towers in NPPs is actually burned uranium, and the smoke from CPPs is "just some smoke". Trust me, there are tons of fools like that, and those are government voters.


I'd really like to see solar thermal gaining more usage. Passive decentralized heating. Of course, where and when it is viable.
For example, here where I live, on the Adriatic coast, we have really hot and arid summers. I'm suffering trough one right now. So why not using solar heaters on roofs? Those things are pipes and black paint, some glass, a boiler and a pump, essentially You get really hot water without any aditional expense.
They pay off in few summers at most. But people are usually lazy as hell, or don't have the money for initial investment. That's where government should jump in. Funny idea, I know. ;)


I see the energy future as a blend of coal/fission/hydro base, gas and wind for the middle and upper middle, and solar (thermal or PV) for peaks and reducing the usage of gas. Coal will not be abandoned, and won't be exhausted soon, that's for sure.


Regarding electric cars, I'm sure you're aware of the fact that power doesn't grow on trees, and it's not "made by wall plugs". Using an electric car only transfers the problem elsewhere. If the base was predominantly fission, and the batteries were a lot better, those cars would be very enviromentally friendly. But in America today, with the majority of power being derived from coal, and batteries being crappy as they still are, they aren't. They only lower the local smog, but the total CO2 production is even higher, because the energy chain gains one more link.

[Edited on 18-7-2011 by Endimion17]

Polverone - 18-7-2011 at 14:56

Quote: Originally posted by Endimion17  

Regarding electric cars, I'm sure you're aware of the fact that power doesn't grow on trees, and it's not "made by wall plugs". Using an electric car only transfers the problem elsewhere. If the base was predominantly fission, and the batteries were a lot better, those cars would be very enviromentally friendly. But in America today, with the majority of power being derived from coal, and batteries being crappy as they still are, they aren't. They only lower the local smog, but the total CO2 production is even higher, because the energy chain gains one more link.


This is not true. In the USA, coal provides just under half of all electricity generated. It is also a decreasing fraction of new generation capacity being added. Even if 100% of electrical energy is supplied by coal, electric or hybrid electric vehicles still have modestly lower lifetime CO2 emissions than pure internal combustion engine vehicles. See Life Cycle Assessment of Greenhouse Gas Emissions from Plug-in Hybrid Vehicles: Implications for Policy

Coal plants can run at higher thermal efficiency than automotive internal combustion engines. Electric vehicles also do not need to keep engines idling while stopped. These factors may account for an electric vehicle "running on coal" still emitting less than a typical gasoline powered vehicle.

I welcome alternative analyses of PV power and electric vehicles from peer reviewed sources, but simply saying "I am very skeptical... I'm not buying it... It can't be..." isn't very good evidence.

gregxy - 18-7-2011 at 14:59

I did a similar calculation and arrived at a similar number. 10,000 square miles of solar cells placed in Arizona to replace the fossil fuels in the USA. A square mile of anything seems like a lot! By comparison there are 3.7e6 square miles of roads in the USA.

Those energy payback times are misleading. That is the time to get back the energy used to create the cells. The more important number is the amount of time to pay for the cost of the cells, which is on the order of 20 years (I live in San Jose CA and the online estimator told me that including a 30% rebate from the government and rate increases from the utilities, it would take it would take 17 years to reach break even). Given that the estimated life of the panels is also around 20 years they may never pay for themselves.

As fossil fuels are depleted their cost will go up and solar will become more competitive and we will see more solar power put into use.

It looks like energy was at its cheapest (in terms of labor expended per joule or BTU or KWh or??
back in the 60s and the price will only go up from here. (Unless there is a breakthrough in Fusion which looks unlikely.)

johansen - 18-7-2011 at 16:27

Its interesting to consider just the electric grid alone.
Wiki states 39KWH per person per day in the USA.

At 5 hours insolation and a currently impossible 20% efficiency to make the numbers easier that's a minimum of 40 square meters of solar per person just for the electric grid alone. 60 square meters is more reasonable.

the solar cells themselves aren't expensive, I recently bought 3 kilowatts worth of 17% efficient cells for $800. its the glass, EVA, frame, shipping and labor to manufacture them that costs money. But if you build them yourself with surplus glass, they will pay for themselves even in the pacific northwest at 3-4 hours of insolation per day.

Endimion17 - 18-7-2011 at 16:43

Quote: Originally posted by Polverone  
This is not true. In the USA, coal provides just under half of all electricity generated. It is also a decreasing fraction of new generation capacity being added. Even if 100% of electrical energy is supplied by coal, electric or hybrid electric vehicles still have modestly lower lifetime CO2 emissions than pure internal combustion engine vehicles. See Life Cycle Assessment of Greenhouse Gas Emissions from Plug-in Hybrid Vehicles: Implications for Policy

Coal plants can run at higher thermal efficiency than automotive internal combustion engines. Electric vehicles also do not need to keep engines idling while stopped. These factors may account for an electric vehicle "running on coal" still emitting less than a typical gasoline powered vehicle.


I stand corrected. Coal is 21.58%, according to EIA in 2009.
http://www.eia.gov/emeu/aer/pdf/pages/sec1_3.pdf

But my biggest concern is all of the fossil fuels. If you combine all of them, you get over 80%. AFAIK, that's how much France produces nuclear power alone.

Could you tell me, cause I don't have the time to study the whole paper, is the life cycle of the batteries included in the study, or just the vehicle alone?


Quote: Originally posted by Polverone  
I welcome alternative analyses of PV power and electric vehicles from peer reviewed sources, but simply saying "I am very skeptical... I'm not buying it... It can't be..." isn't very good evidence.

I'm just expressing my concern, and I base it on my knowledge of energy densities of various sources and their availability.
What might be viable in 20 years for arid Dubai, will never be for a foggy London, no matter what kind of PV we produce.
Because it isn't all in the technology that is "just waiting to improve and become competitive". It's got something in the sheer facts about the sources themselves.
I've got a friend who works abroad in one lab. Thin film deposits and his research is partially in the PV domain. He's one of the most pessimistic people I know regarding light-to-electricity. Cold shower, and I mean freezing cold shower.
That doesn't mean things won't change, but technology is one thing, and the energy source is another one.

I'd be very happy if we had efficient and cheap fusion, and electric cars with very dense energy storage. But we don't.



Quote: Originally posted by gregxy  
I did a similar calculation and arrived at a similar number. 10,000 square miles of solar cells placed in Arizona to replace the fossil fuels in the USA. A square mile of anything seems like a lot! By comparison there are 3.7e6 square miles of roads in the USA.

It is a lot. It's like one third of Ireland!
And it solves nothing. Those are the numbers for PVs running during the day, in the sunny, arid zones, and there aren't plenty of those in the populated areas. When you do more realistic calculations, you get disturbing figures.

But these arguments are not even needed, because no one rational would expect that PV can replace all of the coal. We aren't talking about it either.

What America needs is to finally start respecting the RRR. Reducing the impact and consumption, reusing and recycling what is viable (most isn't, but that's another story). The most important thing is the reduction. All the recycling green hippie movement is utter crap if people don't reduce consumption of resources, and energy is one of them. Yet, we don't see that happening. It's "living the high life" for the most part of the inhabitants, compared to the world.

[Edited on 19-7-2011 by Endimion17]

Polverone - 18-7-2011 at 16:47

Quote: Originally posted by johansen  

the solar cells themselves aren't expensive, I recently bought 3 kilowatts worth of 17% efficient cells for $800. its the glass, EVA, frame, shipping and labor to manufacture them that costs money. But if you build them yourself with surplus glass, they will pay for themselves even in the pacific northwest at 3-4 hours of insolation per day.


Right now the cells themselves account for just over half of system costs, and the balance just under half. It's expected that cells will be just under 50% of system costs by next year because cell costs are declining faster than balance of system costs.

To aggressively reduce solar prices further it is not just the cells that have to get cheaper. It also requires cheaper and more standardized installation permitting processes, and less expensive inverters, mounting, and packaging.

Higher-efficiency cells can kill two birds with one stone. Using higher efficiency cells fits more generating capacity in a fixed area like a single roof. The cells themselves may command a price premium, but can offset that increase by reducing costs for packaging, framing, and installing a kilowatt of capacity. Gradual improvements from process tweaks also allow manufacturers to increase production (as rated by peak watts) without additional capital or labor costs, which means better profit margins for the manufacturer in the short term and lower prices for everyone in the long term.

Polverone - 18-7-2011 at 17:15

Quote:

Could you tell me, cause I don't have the time to study the whole paper, is the life cycle of the batteries included in the study, or just the vehicle alone?

The life cycle of lithium ion batteries is included.

Quote:

I'm just expressing my concern, and I base it on my knowledge of energy densities of various sources and their availability.
What might be viable in 20 years for arid Dubai, will never be for a foggy London, no matter what kind of PV we produce.
Because it isn't all in the technology that is "just waiting to improve and become competitive". It's got something in the sheer facts about the sources themselves.
I've got a friend who works abroad in one lab. Thin film deposits and his research is partially in the PV domain. He's one of the most pessimistic people I know regarding light-to-electricity. Cold shower, and I mean freezing cold shower.
That doesn't mean things won't change, but technology is one thing, and the energy source is another one.


If your friend works in thin film technology I am not surprised that he is pessimistic ;)
For at least 10 years the promise of thin film has been high conversion efficiency combined with low production cost, but it has been very difficult to translate into commercial success. Only First Solar has been a big success, and there isn't enough tellurium available for everyone to imitate First Solar.

Boring conventional silicon PV still has more than 80% of the market, and commercial silicon cells are available with higher efficiency than any commercial CIGS or CdTe thin-film devices. It is hard to tell if thin-film will ever surpass silicon PV in market share. Silicon may already have too large a head start in terms of process experience, manufacturer ecosystems, and economies of scale. It also has the advantage of not requiring expensive and possibly supply-limited elements like indium and tellurium.

I don't think that PV solar can replace 100% of electricity consumption in any currently developed nation. Certainly not in London! But solar cells already have a lifetime positive energy payback even in London's dismal insolation. The question of economic payback is much harder to project. I wouldn't be terribly surprised if PV declines in price over the long term so that it becomes economically attractive even in London (although there isn't enough insolation for London to get a large fraction of total electricity from PV even if it has the lowest marginal cost).

AndersHoveland - 18-7-2011 at 17:34

People keep repeating that solar power can never completely replace coal power. I find this illogical. If solar power can provide 10% of our power needs, is it really that much harder for it to provide 100% of our current consumption? It is merely one order of magnitude difference. Relatively little of the resources in society go toward developing solar power. It would not really be too much of a stretch to improve current efforts twenty-fold. But again, I stress that the best way to reduce our impact on the environment is to reduce our consumption. Better urban designs that would minimize the need for cars (and reduce commuting times) are also important.

Making solar panels more efficient may likely prove increasingly difficult, as incremental improvements take much research, and all the easy strategies have been used. I have read of one design that additionally harnesses the waste heat from the panels using thermoelectric coupling. There is also photothermolectric panels that could utilize much of the short infrared spectrum. This needs more research, but seems very promising. The real savings will come from scale of production; as more solar panels are produced, the price per unit will dramatically fall in the long term (unless future panels require rare-earth elements).

The thing I wonder about is whether, as solar power becomes much more predominant, whether it will become controversial. This has been the case with wind farms. People are complaining it obstructs the natural scenery, and the turbine blades kill many birds. Solar panels on all the houses might be less pleasant to look at. The best place for panels is on the roofs of already ugly commercial blocks and factories, in any case, most of these flat roofs are not even vissible, except from a plane above.

[Edited on 19-7-2011 by AndersHoveland]

Polverone - 18-7-2011 at 18:02

Quote: Originally posted by AndersHoveland  
People keep repeating that solar power can never completely replace coal power. I find this illogical. If solar power can provide 10% of our power needs, is it really that much harder for it to provide 100% of our current consumption? It is merely one order of magnitude difference. Relatively little of the resources in society go toward developing solar power. It would not really be too much of a stretch to improve current efforts twenty-fold. But again, I stress that the best way to reduce our impact on the environment is to reduce our consumption. Better urban designs that would minimize the need for cars (and reduce commuting times) are also important.


I think you answered your own question. Solar will not provide energy equal to 100% of current consumption not because it is physically impossible, but because A) it is not going to be the best option everywhere, even among renewables and B) even if you're giving up all fossil fuels, it's cheaper to economize on consumption first than to build out non-fossil energy sources as a 1-for-1 replacement of oil, gas, and coal.

I think that solar PV has a better shot than competitors about overcoming NIMBY opposition. It doesn't make noise, it's not visible from far away, and in normal operation or failure it doesn't pose any great health risks to nearby animals* or humans.

Solar and wind have the additional unfortunate problems, mentioned earlier, of being intermittent and at least somewhat unpredictable. This means that you cannot continue to operate a stable electrical grid with a large wind/solar component without building in storage mechanisms or excess dispatchable generating capacity, which greatly increases the cost. If someone invents an inexpensive, rapidly responsive, non-geography-dependent storage mechanism for renewable energy I'll go from cautiously optimistic about solar to downright bullish. But in discussing the future potential of solar or other energy sources I don't like to assume any big breakthroughs.

*At least if it's deployed on roof tops or brownfield sites. Deploying it on large scale in remote areas may indeed harm animals due to the ground shading, new transmission line corridors, and new access roads.

Endimion17 - 19-7-2011 at 01:03

NIMBY's are the least concern. If you spend some money on education of people, they will understand and won't make a fuss out of it. But if you let them live in ignorance and fear, you get the situation with fission in America, where towns use old power plants from the 70's instead of building passive safety, new generation reactors like France does (and earns a whole lot of money by selling the energy to dumb Italy which halted its nuclear power program because of fear, like radionuclides from France couldn't ever cross the border... dumbasses).

PV can't replace coal in the most of populated places in Northern Temperate Zone because the energy source is unpredictable and has extremely low energy density.
And it doesn't work at night. People seem to forget that their efficiency, which is poor and today reaches almost 30% for the most expensive and newest and best laboratory samples (the ones on the market are usually 10% if their wafer quality is good) is only when the Sun is high in the sky and there are no clouds. And that is not the case. In real life, you have clouds, low Sun, atmospherilia gradually eating the panels and greatly reducing their lifetime, dust, etc.

And each kW of PV requires a kW of gas. Because making so much batteries that could provide a whole town/country the power at night is ... I don't know how to express myself properly... ludicrous. :D
Batteries are things for emergency backup for some devices and machinery untill the diesel generators kick in. Not for supplying the country during the night.

PV can not be an alternative source. Alternative means "to replace with", and they can not serve that purpose.
PV can only be a substitute and yes, I agree with using them as a substitute where and when viable in energy, economy and enviromental sense.
They are complementary energy sources and can not be the base od the power system. Ask anyone working in the energetics. They'll laugh at the idea of PV base system. Not because they're dumbasses with a habit, but because they understand the problems which are way more complex than we talk about here and deal with power grid stability (wind farms wreck havoc with power grids!). It's not just plug and play. People at electricity distribution centers work their asses off in winter and especially summer.

One of the biggest problems with PV, if you ask common folk that doesn't concern with energy and enviroment viability is economy. I don't know much about it, but as I've heard it's problematic.

froot - 19-7-2011 at 01:38

Quote:
If someone invents an inexpensive, rapidly responsive, non-geography-dependent storage mechanism for renewable energy I'll go from cautiously optimistic about solar to downright bullish.


This is in my opinion the crux and the bag of sand tied to the bicycle that's trying to move towards renewable energy.
Even new uncommercialised energy storage technologies of today still have discouraging drawbacks that restrict them to someone's lab. Efficiency, longevity, robustness, cost, capacity, intricity, toxicity, etc are a few. I've spent hours reading about VRB's, Zn/Br redox, metal air, NaS batteries, NiH cells, K-ion cells, a spectrum of fuel cells, flywheels, supercaps, and whatever else I cannot recall now and all of these feature at least one nasty drawback, especially on a homeowner's level with PV's on the roof. It's a problem. We have environmental energy capturing appreciably sorted, we just don't have anywhere to effectively put that energy. Running a grid directly without load levelling is not an option either.
The most attractive energy storage technology I've found to be possible a candidate is the NiH battery (the pressurised type) with a longevity of over 10 000 cycles, far better than anything else out there, and it's not even new.

Twospoons - 19-7-2011 at 14:11

I've always thought the first step for introducing solar should be domestic water heating. Its more efficient than PV's, and the water cylinder provides inherent energy storage. The tech can be anything from painted black pipes to mirror enhanced, vacuum encapsulated heat pipes with selective absorber coatings.
Heating water takes a lot of energy - taking that load off the grid makes sense to me.

I don't think there is a magic bullet for renewable energy - it is going to take a wide mix of technologies to create reliable energy supply, including fossil and nuclear sources. The trick is to change the balance from 10% renewables to 90% or more.

[Edited on 19-7-2011 by Twospoons]

497 - 21-7-2011 at 17:16

Come on guys. Just let the plants do the energy capturing for us. They've been fine tuning it for billions of years. And then you don't have to figure out how to power the transportation sector with electricity.

Switchgrass will grow damn near anywhere in the country, make at least 10 dry tons of biomass/acre/year. The energy input needed to plant and harvast it is near 5% of the energy captured. And it uses already mature hay processing technology. Eventually algae may end up being more productive, especially with direct capture of CO2 produced from various stationary sources.
Plasma gasify that shit (along with all the other collectable organic waste in the country) to high purity syngas. Conventional gasifications works too, but it seems really tough to get the tar production to a manageable level. From there you can generate electricity via IC engines, turbines, or fuel cells. Transportation fuel can be produced simply by catalytic reactions to form hydrocarbons through the Fischer-Tropsh route, or methanol (which has the advantage of being directly usable in fuel cells) along with all the other petrochemicals that we use to make plastics, etc. Hydrogen could be produced in high yield from syngas via the water gas shift reaction if you'd rather deal with the headache of hydrogen storage/transport.
All but the plasma gasification is well proven technology..
Ideally a relatively simple and cheap system could be developed using easily available parts that would allow fully decentralized production of liquid fuel, electricity and heat directly by anyone with a source of cheap biomass. Can't wait to see how the oil companies react to that! :D It seems at least somewhat possible since the entire (microwave) plasma gasification --> Fischer-Tropsh reaction can be performed at atmospheric pressures, and no expensive catalysts are required. You might even be able to decentralize the needed nitrogen fertilizer production using microwave plasma reaction of air to form NO..
I think the only way we're ever going to pay reasonable prices for energy is to decentralize it. That's the only way I can see to get real competition. The oil companies have a monopoly that they're not going to give up easily. The only way is to force them by making the technology available to a large number of people. The government is never going to help that happen either, so it's pretty much up to us..


[Edited on 22-7-2011 by 497]

Polverone - 21-7-2011 at 18:27

I agree that biomass will have a valuable role to play as feedstock for chemicals and aviation fuel, but for replacing fossil fuel electricity or automotive fuels it's lackluster unless you have a very low population-to-land ratio.

Plants aren't very good at converting solar energy to human-usable forms. You can get about 188 gigajoules (thermal) of energy back out of that acre's worth of switchgrass per year. Maybe 85 gigajoules electrical can be realized, if you burn it in a high performance supercritical steam turbine system. By comparison, an acre of 15% efficiency solar panels will yield 3980 gigajoules of electrical energy per year at average insolation in the USA. You can also site solar panels where switchgrass won't grow, like in deserts and on roof tops. Switchgrass gets some credit for providing its own energy storage system and for being cheaper to reproduce than manufactured panels, but not enough (IMO) to offset the 20 to 45-fold efficiency advantage of artificial systems.

497 - 21-7-2011 at 23:57

Good points. I did fail to emphasize the more important source of biomass: algae. Grow it in dirt cheap polyethylene bag photobioreactors on the ocean or anywhere else. 20-40 dry tons/acre/year is a reasonable estimate. Uses wastewater as nutrients and CO2 from power generation.

You also didn't mention how much an acre of solar panels costs.. From some quick googling it looks to be on the order of $3-6 million initial investment per acre plus maintenance. Then you must have a massive battery bank or other storage system to allow a smooth supply of power when its not sunny. And all with only a 30 year lifespan.. Additionally you didn't take into account the problems of providing transportation fuel from the electricity. A biomass source that can be directly converted to a liquid fuel usable by current vehicles seems to be very valuable thing.

Until someone figures out a good way to build your own PV panels, all those millions of dollars per acre will keep going to supporting the monopoly.

So I'll have to humbly disagree on this one. I'd much rather spend a thousands of dollars to outfit my acre of land with my home built photobioreactor-plasma gasifier system which could provide me with heat, power, fuel for my car, and potentially a supply of highly nutritious animal/people feed, than spend a few million on solar panels and batteries that will give me a bunch of electricity that I could most likely buy for around 12 cents/kwh.. And the damn utility companies won't even buy your power for that much in many places.

Edit:
After doing more research into algae I have found several very promising new ways to extract the valuable materials from algal cells. This patent basically runs the algae through an electrolysis cell to burst their cell wall and allow easy separation of the lipids in a flotation tank. Avoids the headache of filtering and drying the algae.
Even more interestingly, I found ways to extract goods out of algal cells without killing them. This obviously allows much higher production rates than traditional destructive extraction methods. They look quite viable for scaling down for use in decentralized systems. Basically you extract the compounds of interest from the living cell with a biocompatible immiscible solvent. Dodecane, etc is used. Kerosene should work :D Exciting stuff! This is a very detailed and long winded paper about extraction of beta carotene from algae. This is a patent based on the same principal, but directed toward lipid extraction rather than high value chemical extraction. They also figured out that sonocating the algae/solvent mixture was more effective than mechanic mixing. Thisis another interesting patent that details effective ways of cycling the conditions of an algal culture to maintain the dominance of the desired species in the presence of competitors, particularly in open pond style cultivation.

[Edited on 22-7-2011 by 497]

Paddywhacker - 22-7-2011 at 03:05

Here is a nice large passive and cheap solar energy collector ... http://www.gizmag.com/enviromission-solar-tower-arizona-clea...

The thing about biomass is that currently we are, directly or indirectly, eating most of what we can produce, and I don't think that is going to change much in the near future.

Also, there is a little confusion about the source of CO2. The problem is with burning fossil fuels. Carbon that has been locked away from the biosphere for geological ages. Once in the biosphere it circulates between biomass and CO2 with a relatively short half life. Biomass, forests, algae, animals .... they are all just temporarily solid carbon. Part of the biosphere pool. It is the continual additions to that biomass/CO2 pool that are not reversible and that are of concern. So South Americans burning ethanol are not doing anything to the balance, but Shell Oil and their ilk and coal mining are another matter.

497 - 22-7-2011 at 03:28

Yeah I can't deny those passive towers are a cool idea. I still don't like the fact that it must be done on a massive multimillion dollar scale though.

The point is not to utilize fossil fuel CO2 (though that would be inevitable in the short term), but to utilize the very same CO2 that you get from burning the algae or other biomass to grow more algae. It just makes things more efficient because the algae can grow more rapidly with a high concentration of CO2. It makes no sense to dilute it with huge amounts of air and then make the algae work harder to extract it back out.

bbartlog - 22-7-2011 at 04:54

Quote:
Come on guys. Just let the plants do the energy capturing for us. They've been fine tuning it for billions of years.


Unfortunately, their goal has not been to optimize energy storage efficiency; thus the efficiency of conversion to storage is not really very high.

Quote:
And then you don't have to figure out how to power the transportation sector with electricity.


Doesn't look like a whole lot more figuring out needs to be done. We have electric cars that work pretty well, with further advances in the pipeline.

Quote:
Switchgrass will grow damn near anywhere in the country, make at least 10 dry tons of biomass/acre/year. The energy input needed to plant and harvast it is near 5% of the energy captured.


What about the other inputs (NPK)?

Quote:
Eventually algae may end up being more productive,


Algae production of fuel will never be economical. The efficiency is too low, avoiding contamination by competing organisms too hard, and the workup (for want of a better term) to extract the fuel and recycle the nutrient matter is a PITA (at least economically).

Quote:
Plasma gasify that shit (along with all the other collectable organic waste in the country) to high purity syngas.


Nothing says 'easy and economical' like bringing plasma into it!

Really, you seem to know a lot about the processes, the chemistry, and the industry behind all this, but I don't think you've looked at the economics hard enough.

Endimion17 - 22-7-2011 at 05:22

Quote: Originally posted by Paddywhacker  
Here is a nice large passive and cheap solar energy collector ... http://www.gizmag.com/enviromission-solar-tower-arizona-clea...

The thing about biomass is that currently we are, directly or indirectly, eating most of what we can produce, and I don't think that is going to change much in the near future.

Also, there is a little confusion about the source of CO2. The problem is with burning fossil fuels. Carbon that has been locked away from the biosphere for geological ages. Once in the biosphere it circulates between biomass and CO2 with a relatively short half life. Biomass, forests, algae, animals .... they are all just temporarily solid carbon. Part of the biosphere pool. It is the continual additions to that biomass/CO2 pool that are not reversible and that are of concern. So South Americans burning ethanol are not doing anything to the balance, but Shell Oil and their ilk and coal mining are another matter.


Solar towers cheap? In what universe? :D
Do you have any idea how hard it is to build huge structures? That tower is twice the size of ESB.

No way, Jose.
Honestly, I thought that majority of people on this forum will understand the problems with renewables, becuase everyone si "sciency" and all that, but it seems to me that the situation is only slightly better than on other, general purpose forums.

Solar towers are a stupidity collosal almost as their size, rejected and laughed upon in the year following their popularity spike on the web, few years ago. I mean, solar molten salt with those reflectors is very expensive, let alone a big ass concrete tower with a huge hermetically sealed area. No one would want to pay for such expensive electricity, and the cost return time for money and energy would be incredible.

200 MW for 0.75 billion dollars? You can bet the price will jump over the 1 billion $.
11 years of return? This is a scam. OMG don't be naive.


The thing with energy is - we're fucked. Plain and simple. There's no magic solution, there're no "zero pollution" sources. And we have to deal with it, avoiding to fall into scams about certain recycling that in the ends pollutes even more.

[Edited on 22-7-2011 by Endimion17]

Polverone - 22-7-2011 at 09:43

Quote: Originally posted by 497  

Until someone figures out a good way to build your own PV panels, all those millions of dollars per acre will keep going to supporting the monopoly.

So I'll have to humbly disagree on this one. I'd much rather spend a thousands of dollars to outfit my acre of land with my home built photobioreactor-plasma gasifier system which could provide me with heat, power, fuel for my car, and potentially a supply of highly nutritious animal/people feed, than spend a few million on solar panels and batteries that will give me a bunch of electricity that I could most likely buy for around 12 cents/kwh.. And the damn utility companies won't even buy your power for that much in many places.


You can build a few-tons-per-annum plasma gasifier coupled to a Fischer-Tropsch synthesis system and miniature refinery all for under (say) 20,000 dollars? If you're right I will gladly eat my humble pie and come back for seconds. I'll be too excited that you were right to be sad that I'm wrong. But I don't think that this is realistic in the near term.

In the long term I can picture certain advancements that might make it practical, but in the long term I also think PV and electric vehicle prices will fall considerably, so I'm still not sure biomass is the most affordable way to go.

gregxy - 22-7-2011 at 10:37

I though bioconversion made sense since the collectors manufacture themselves particularly if floating ponds on the ocean could be used.

Solar conversion efficiency for algae is claimed to be 7%
http://biofuelsdigest.com/bdigest/2011/02/18/peer-reviewed-a...
Although it does not say if this includes all the energy consumed to grow and process the algae.

The grass miscanthus is claimed to have the highest conversion efficiency at 1.3 to 2%.
http://abstracts.aspb.org/pb2006/public/P02/P02017.html

In terms of efficiency these compare poorly to PVs for an electric economy.

Oil produced from algae is estimated to cost $8.00/ gallon
as opposed to $4/gallon for soybean oil (seems like the algae oil should be cheaper?)
http://www1.eere.energy.gov/biomass/pdfs/algalbiofuels.pdf
as opposed to ~$2/gallon for crude oil.

smuv - 22-7-2011 at 21:56

How about coal to liquids? Anyone have a figure for how much it would cost to produce straight alkanes? How about reformed to gasoline?

497 - 22-7-2011 at 23:11

Quote:
We have electric cars that work pretty well


But how long to the entire world is using them? I can't deny that eventually that may be a good solution, but right now, its not the case.

Quote:
What about the other inputs (NPK)?


Actually very small compared to food crops. And where do you think all that ash from gasifying it would go? In the case of algae, the issue is even smaller since there is plentiful municipal and agricultural wastewater to be used. In fact the savings on water treatment can be large, potentially rivaling revenue from the fuel/power produced.

Quote:
The efficiency is too low, avoiding contamination by competing organisms too hard, and the workup (for want of a better term) to extract the fuel and recycle the nutrient matter is a PITA (at least economically)


In years past these things were true. Very recently, major advances have been made. Efficiency may be "low" but it is quite a bit higher than any plant. Effective high throughput low energy methods to selectively kill unwanted organisms have been developed. And as for the the workup, yes the traditional filter/centrifuge-press/solvent extract method was quite uneconomical, but there is a new way. You continuously suck the algae slurry out of the main reactor/pond, mix it with about an equal volume of kerosene, sonicate or stir it, allow the layers to separate, return the algae slurry to the main container, and you're left with a kerosene solution of quite pure triglycerides. The triglycerides can be removed from the solvent by various means such as absorption, distillation, or membrane reverse osmosis. The algae growth is unharmed and sometimes improved by this process due to removal of trace toxins that would normally accumulate. A process like this has been recently published to produce b-carotene at about 25 times the areal productivity of the only currently commercially used process, while requiring no handling or drying of concentrated algae, only the normal culture solution and a recyclable ~$6/gal solvent is handled. As far as I can tell no energy input other than pumping, briefly sonicating and separating the resulting solvent/algae oil solution is needed. Obviously these results were not directly applicable to algae oil production, but the do at least give an idea of possible improvements that can be made by the process, not to mention improvements from genetic engineering research which seems to be just beginning.

Additionally, the ability to grow algae on the oceans surface is a great advantage.

Quote:
Nothing says 'easy and economical' like bringing plasma into it!


When you realize that bringing plasma into it means a free microwave magnetron, some welded steel and hi temp furnace cement, yes it can be easy and economical. Especially when you get a gas out that requires minimal purification, versus the extensive multistep purification methods required for making usable syngas with a conventional gasifier.

Quote:

How about coal to liquids? Anyone have a figure for how much it would cost to produce straight alkanes? How about reformed to gasoline?


Cheap and totally doable in my opinion. Do people have actual evidence that it would not be way cheaper? Probably not as profitable to those who call the shots though. Not as preferable for pollution reasons too. Still shouldn't be any dirtier than the current system, maybe somewhat better even. It is also much less doable on a small scale, due to the coal mining and pollution issues probably. Plus the fact that economies of scale seem much more potent when dealing with coal.

Quote:
You can build a few-tons-per-annum plasma gasifier coupled to a Fischer-Tropsch synthesis system and miniature refinery all for under (say) 20,000 dollars?


I really can't blame you for being skeptical. But after a bit of research (much more still needs to be done) I have come to the conclusion that it is doable. Not saying it is for everyone, because I have access to highly skilled welding (stainless steel and aluminum too) along with many other common metal fabricating tools drills, compressor, cutting tools, a small lathe, etc for free nearby, and a source of at least 6-10 horses worth of manure+bedding on site as a feedstock (which must otherwise be removed by spending money) and a small bobcat loader on site for manipulating it. Also there is larger heavy equipment such as dump trucks around for nearly free use to potentially bring in other biomass, especially the nearby available sawdust from a sawmill.

10 tons per year is less than 3 pounds per hour. That is easily within the capabilities of one or two microwave magnetrons obtained for free. The plasma gasifier is not complex, just a simple adjustable waveguide (which I admit may require much trial and error) focusing the microwaves into a small perlite+furnace cement insulated chamber into which is fed (preferably preheated) biomass along with uncondensable gases coming from the FT reactor. The ash forms an easy to handle hard vitrified slag (versus the sticky powdery shit that conventional gasifiers make). The hot output gas from the chamber would be directly and very rapidly cooled by passing through a water fed venturi. The water may contain additives to help remove inorganic contaminants, and possibly CO2. Alternatively a second CO2 scrubber may be used. Then pass the gas into a condenser to remove most of the remaining water vapor. Then a heat exchanger using waste heat from the gasifier would reheat the gas to the appropriate temperature for FT conversion, about 250-300*C IIRC. Before the FT reactor it would pass through a canister of dirt cheap ZnO to scrub remaining sulfur. The FT reactor would most likely consist of a 2-4" dia vertical pipe and smaller downcomer tube. The tube is filled with a mineral oil suspension of iron particles made by reduction of dirt cheap iron oxide. The gas will be bubbled into the bottom and and after exiting the top will be passed into a condenser. If the temperature and flow rates in the FT reactor are right, the condensed liquid will be directly usable as gasoline without further processing. The unreacted syngas along with some methane/ethane/propane will be brought back into the gasifier (or used to run an stationary IC engine for example).

Only minimal syngas purification is required because a correctly running plasma gasifier will produce only nanograms of tar per m^3 of gas, versus a conventional gasifier that is tough to get below the grams/m^3 level, thus requiring vastly more expensive and complex purification to meet the requirements of the FT catalyst.

No pressure vessels required, unless you count a moderately pressurized cooling water feed.

What part of that looks expensive to you?

10 tons per year @ 50% moisture (for horse manure+bedding mixture, but any other biomass will work as well) = about 4500 pounds of C. At even 50% efficiency of conversion that will result in about 325-375 gallons per year, equivalent to $1300-1500 at current prices, which will undoubtedly be higher by the time its running. Since I live in a cold climate, all of the waste heat from every aspect will be used for building heating and will displace a very substantial amount of fuel oil that costs $3/gal. The waste heat may be reclaimed another way in warmer climates.

Quote:
Oil produced from algae is estimated to cost $8.00/ gallon as opposed to $4/gallon for soybean oil (seems like the algae oil should be cheaper?)


The past processes used to extract that algae oil were very inefficient and uneconomical. With newer methods the price can be dropped a long way. Exactly how far, I don't know yet. Hopefully we'll find out soon. Also, ocean surface algae production has yet to be developed much, as far as I can tell. . I'm guessing that estimate was not based on ocean surface production. With additional advances that require less "sterile" conditions, the cost of growing the algae should drop too. Imagine if relatively untreated seawater (maybe brought up from deep nutrient rich waters) could be used as a growth medium? Thin polyethylene sheet is pretty damn cheap. I've seen designs (untested yet as far as I know?) that consist of almost entirely of polyethylene sheet and various sheet plastic check valves, and plastic filter membranes etc at least up to the point of processing the algae cake or possibly transporting the crude algae oil. They used wave or tidal power to do all the work for pumping fluids around. They were probably drawn up before the non destructive extraction process was known. Combining that with the wave powered pumping, etc could be very attractive. Simple electromagnetic wave powered generators could be integrated to produce power needed to do various things including separate the solvent algae oil mixture at sea. The algae farm would have the capability to inflate/deflate floats (either holding less saline water or air) to sink below the surface in the event of a storm/boat. Tanker/pipe the crude oil to shore, maybe along with pretreated wastewater out and/or treated fresh water in, you get the picture. The farms could even double as rain collectors under certain circumstances.

Hah shit, you know I bet you could even use the finished biodiesel product or a close derivative of it as the solvent!

Yes there could be regulatory barriers. I'm pretty sure they could be dealt with, with proper design, and regulation in a responsible manner. I seriously doubt anything nearly as catastrophic as a large oil spill could occur, barring the release of an invasive genetically modified algae. Obviously serious steps need to be taken to prevent that from happening.

It seems to me that best way for truly rapid innovation to occur in an environment where very very powerful people want slow it down is to take it out of their hands. Out of the secrecy of the corporate and university labs that in one way or another are more concerned with keeping the oil company's good graces or getting rich of it. The only way for that to happen is to develop a process that is simple and small scale enough to be done by the individual. This is a challenging task, but one I believe may be vital to our futures if we want to maintain/improve our standards of living. Plant crop biodiesel and fermentation routes are a good start, but they're not going to go far enough for one reason or another. Some solar and hydrokinetic based methods like interesting and maybe eventually viable, but they seem much more thoroughly studied that algae based methods.

Hopefully now you can understand why I am not basing my statements on the pure large scale economics that many others rely on. Sorry for writing so much..

[Edited on 23-7-2011 by 497]

Polverone - 23-7-2011 at 10:11

Quote:
10 tons per year is less than 3 pounds per hour. That is easily within the capabilities of one or two microwave magnetrons obtained for free. The plasma gasifier is not complex, just a simple adjustable waveguide (which I admit may require much trial and error) focusing the microwaves into a small perlite+furnace cement insulated chamber into which is fed (preferably preheated) biomass along with uncondensable gases coming from the FT reactor. The ash forms an easy to handle hard vitrified slag (versus the sticky powdery shit that conventional gasifiers make). The hot output gas from the chamber would be directly and very rapidly cooled by passing through a water fed venturi. The water may contain additives to help remove inorganic contaminants, and possibly CO2. Alternatively a second CO2 scrubber may be used. Then pass the gas into a condenser to remove most of the remaining water vapor. Then a heat exchanger using waste heat from the gasifier would reheat the gas to the appropriate temperature for FT conversion, about 250-300*C IIRC. Before the FT reactor it would pass through a canister of dirt cheap ZnO to scrub remaining sulfur. The FT reactor would most likely consist of a 2-4" dia vertical pipe and smaller downcomer tube. The tube is filled with a mineral oil suspension of iron particles made by reduction of dirt cheap iron oxide. The gas will be bubbled into the bottom and and after exiting the top will be passed into a condenser. If the temperature and flow rates in the FT reactor are right, the condensed liquid will be directly usable as gasoline without further processing. The unreacted syngas along with some methane/ethane/propane will be brought back into the gasifier (or used to run an stationary IC engine for example).

Only minimal syngas purification is required because a correctly running plasma gasifier will produce only nanograms of tar per m^3 of gas, versus a conventional gasifier that is tough to get below the grams/m^3 level, thus requiring vastly more expensive and complex purification to meet the requirements of the FT catalyst.

No pressure vessels required, unless you count a moderately pressurized cooling water feed.

What part of that looks expensive to you?


If you want to spend no more than 3 hours per day operating the system, and it doesn't look like it can operate unattended for long, you need to process more like 20 pounds per hour (figure some time is spent on maintenance).

My skepticism is mostly a matter of scale. If you proposed to do bench-scale demonstrations with grams of material for any of the stages proposed, I wouldn't think you were unreasonable. To do it on a scale of even a few tons per year, with only one operator working a few hours a day, and with no online analysis to check product output or syngas H2:CO ratios, it seems that there are a lot of ways it could fail to meet expectations. But if you pull it it off, I will be very impressed.

[Edited on 7-23-2011 by Polverone]

497 - 23-7-2011 at 11:03

I've read quite a few papers where their bench scale model was in fact using pounds per hour, sometimes more than 3 even. The apparatus can afford to be pretty small when the residence time is very small.

I'm not really sure what you think the continuous attention would be needed for.. A hopper and auger feed are not a tough thing to accomplish and automate. Neither is an slag auger. Thermostatic control of heat exchangers and cooling water pumps is pretty doable. Manual feed hopper refilling, slag bin emptying, ZnO canister changing, and possibly partial catalyst exchanging can all be done on a weekly basis. I don't see why online analysis of product output and uncondensable gas output is that tough. Sure H2:CO ratios are harder to analyze, but they're not really critical for a high temperature FT reactor from what I've read. The unreacted gas will be utilized too. If the ratio was consistently low (the most likely scenario) it wouldn't be tough to add an iron catalyzed water gas shift reactor to improve it. I won't be the only one around to operate it either, especially after the initial period of optimizing its settings.

I think people underestimate how well these processes work on a small scale because that's just not the way they're ever run. I don't think it means they can't be done. How much effort do you think has ever even been put into scaling them down? In fact I know that some processes the FT reaction for example actually worked better on a bench scale and more problems were encountered in scale up.

gregxy - 23-7-2011 at 14:18

http://www.peat.com/pdtr-100.html


Here is a system that you can buy (130lbs/hour). It is targeted at disposing of
medical waste using a plasma torch. It does not appear to generate any net
power (130kW + CH4 + water + N2 in, about 25kW out).

Some of these systems do claim to generate net power, but it must be highly
dependent on what you feed it. For something like rubber tires yes. For damp
biomass I don't see how it could generate power given that you can't
burn the stuff.

It seems like a big problem would be that a large amount of power must
be cycled back to feed the plasma torch. Converting the syngas to electrical
can only be done at low efficiency without multi stage gas/steam turbines.



497 - 24-7-2011 at 01:16

Quote:
Some of these systems do claim to generate net power, but it must be highly dependent on what you feed it. For something like rubber tires yes. For damp biomass I don't see how it could generate power given that you can't
burn the stuff.


Microwave induced plasma tends to be can be considerably more efficient. Here is an example. On that page they also adress the water content issue:

Quote:
“One of the main differentiating aspects of our technology compared to plasma arc or torch technologies is that it utilizes water contained inside the waste, where the water actually becomes part of the chemical reaction to produce the syngas,” Tijerina said. “We can bring damp waste into our reactor and in some cases we might even inject water to increase the conversion yield of the waste.”


It seems like you guys are using a lot of information about something conventional to judge something that is unconventional. Not surprising since since it is the widely available and well known information..

Woo 497 posts!

[Edited on 24-7-2011 by 497]

watson.fawkes - 24-7-2011 at 06:30

Quote: Originally posted by 497  
What part of that looks expensive to you?
The high-nickel alloys needed to deal with hydrogen embrittlement. They're not needed to make a short-lifetime prototype, though.

497 - 24-7-2011 at 13:04

Considering the system is not under pressure, I find it unlikely that 304 stainless would be weakened enough to fail. Very low loads are involved. From what I understand the parts of the system that were maintained at higher temperatures would not be nearly as susceptible. Further analysis is surely waranted. Even if more expensive allows were required, on a small scale I doubt would be anywhere near prohibitively expensive. Especially if copper or nonmetallic parts can be used for some purposes. According to this reference:

Quote:
Historically, carbon steel or stainless steel has been used to transport hydrogen. Gray, ductile, or cast iron and nickel steels have been used but are not considered suitable for highpressure hydrogen transmission (Mohipour et al. 2004). Austenitic stainless steels, aluminum (including alloys), copper (including alloys), and titanium (including alloys) are generally applicable for most hydrogen service applications. High-strength steels (above 100 ksi) are more susceptible to hydrogen embrittlement, so the use of thicker, low-strength steels is sometimes recommended for hydrogen pipelines. Polymer/fiberglass-reinforced pipes have been used in specific applications such as for in-plant piping at moderate temperatures.


Thor - 26-7-2011 at 08:46

Having done the algae experiment myself a few years ago. Its not very efficient. I grew a large batch of algae, in artificial conditions, high light levels, high nutrients and then extracted the lipids with Heptane. For the amount of time it took and the volume of algae the yield was pretty pitiful. To then use this in an inefficient IC engine is madness. Electric cars are now in mass production, and i believe are the future.

gregxy - 26-7-2011 at 11:06

I remember reading in a study that the amount of lipids produced is higher when there is insufficient nitrogen.

There was a DOE study on algae farming back in the 80s
which is described in the following report.

http://www.nrel.gov/docs/legosti/fy98/24190.pdf

smuv - 26-7-2011 at 22:49



Gregxy, that is a really interesting report.

I am surprised by the response to this thread. It seems a lot of people on sciencemadness are following renewable energy sources, maybe a sticky about green/alternative energy sources would be a good idea? Not only would it be fun to read, it would be nice to have a consolidated source of information about emerging energy technologies.

...just a thought...

Dr.Bob - 3-8-2011 at 07:30

After reading a lot about alternative energy and energy conservation, I have to agree that cutting consumption is a good start (insulation, higher fuel economy, more efficient furnaces, less waste of power on lighting, computers left on all night, other obvious waste), then simple things like solar thermal hot water, geothermal heat pumps, LED lights, hybrids, etc are a good second step. And lastly, alternative energy (PV, windmills, tidal systems, biomass) and all of the fancy stuff that is expensive and hard to implement. If every house in America was just insulated, had a moderately efficient HVAC system installed, used Energy star appliances, low power lighting, got one more energy efficient vehicle, and some small amount of common sense was used, we could cut energy use in America by 30-50%, with almost no change in living standards. In fact, several of my friends have done it, and saved money within a short time, through insulation, geothermal heat pumps, better hot water systems (the new heat pump designs work well), and other small changes. Most of them will have a payback of 3-5 years at most.

Solar PV works best in places with 1) high electric costs, 2) good sun (southwest US, Hawaii, Florida) and 3) good tax breaks. So places like the west coast of the big island of Hawaii are great choices, due to high electric rates, poor grid and high costs of wiring infrastructure, and year round sunshine. Southern Cal. also is a quick financial payback.

Some of the best tax breaks in places like NJ where the amount of sunlight is not optimal, seem like a waste, especially given that solar panel production is one of the limiting factors in PV usage. I would rather see solar PV used first in places like Arizona, NM, Cal, Colorado, Texas, etc, where they give the most return of energy. As long as they are under 10% of the total energy use, the grid can buffer their production as the sun rises and sets and clouds move.

As for biofuels, I see them as a way to stretch our current petrofuels, not a replacement. if we could find a way to provide a crop that would generate 4 times more oil per acre than soybeans (about 10% usable weight of the crop is oil), then we could make a serious dent in the amount of diesel used, and eliminate the need for imported oil in that fuel sector. Since other crops in tropical areas provide way more oil per acre, I think the problem is a solvable one. Combine that with more productive ethanol production (we need to improve the energy input:output ratio to about 1:3 or 1:4 to be reasonable) and other ways to produce liquid fuels, and we can reduce our oil imports by a sizable margin. Coal to liquid fuels/gasoline has been done for years, is still
done by Eastman Kodak and SASOL today, and is financially reasonable at the current oil prices. That will eventually become another source of fuels.

And lastly, nuclear will have to continue to be part of the energy use on earth if the population keeps growing, as no other energy source is so scalable. I think the use of smaller, modular reactors, used in parallel, might be one answer, almost like a bunch of nuclear submarine sized reactors all put on the grid. In fact, those would be great , as they could be designed to work underwater in case of disaster. :-)

So our energy problems come about from several sources: growing population, higher standard of living world-wide, more dependence on energy intensive processed for raw materials, wide use of computers, electronics, cars and refrigeration, global warming potential, peak oil production, pollution, nuclear woes/fears, NIMBYism, ecological desires not to damn every river on earth, soccer moms driving 3 ton SUVs, etc.

So we need to work on parallel approaches to the solution, as I don't see any one answer to the problem. But if everyone even did the options that have short paybacks, and built new homes to higher energy standards, the problem would be reduced substantially. Habitat for Humanity here builds only super efficient homes, and the building costs only rise by a small amount, in part due to the ridiculously small HVAC systems needed, and they average $40 per month for all of the heating, cooling and hot water. So it is easy to do, and affordable.

Bob

497 - 9-8-2011 at 05:02

The culture of consumption is far too entrenched (and spreading) to allow much of the above to happen in a timely manner. To me, the possible outcomes look to be either the system runs itself into the ground and needed lowering of (percieved) standard of living is forced by the resulting catestrophic global reprocussions, or a profitable solution that maintains the general status quo is developed (with extra green colored packaging for piece of mind.)
If you ignore the irrationality and greed of humans there is a plethora of suitable alternative energy sources. But how many will work in the real world?
When so much human energy/will is devoted toward extracting money from their peers, perceptions become prioritized much more than efficiency, practicality or sustainability. The magnitude of this is still hitting me as I spend time in urban areas.
Efficiency and percieved socioeconomic status do not mix (past buying organic food and hybrid cars anyway.) To change this would require a direct reversal of the philosophies of the people who have the least to gain, the most to lose, and ever increasing power to manipulate the public's perceptions. Not likely to occur by choice, but one way or another it will happen. We can only hope for the ability to choose how..

As for nuclear, why the hell can't it be switched over to a gas cooled pebble bed style reactors? They seem so much more trustworthy with the bonus of being much harder to weaponize.. With recent events I really can't blame people for being wary though. I doubt it will ever be much of a solution to our energy problems.

asilentbob - 17-8-2011 at 21:01

Consider that the sun is a large fusion energy source at an ideal distance away from the earth...

A very large portion of the energy we use on earth is from the sun already. Granted some of it like from oil has been captured by plants into chemical bonds, eaten by animals, and so on, decomposed and reformed, then drilled, pumped and refined... Oil is solar energy in one chemical form. With so many steps its not as efficient as solar panels or solar thermal uses, etc... But its easy at the moment. Any time someone says that oil is more efficient than solar photovoltaic... they don't get it. They are looking at oil as if it has always been here without thinking of how much energy it took to create it.

We think of solar energy as a pipe dream because we have been living off the oil from so long ago. We just pump it out, refine it, and burn it. We only see half of its life. You can think of the oil reserves on earth as a sort of rechargeable massive battery which was very inefficiently charged up by solar energy long ago... and which we are beyond the half-way point of draining, using very inefficient machines. A capacitor may be more accurate since we have been draining it so quickly... If we never had the oil to use, we would most certainly be using solar panels, solar thermal, nuclear, wind, etc... And these types of threads would not exist. If life would be around at all.

Solar voltaic, solar thermal, etc... are solar energy like oil, but more direct and more efficient. As are grown crops. The food you eat (hopefully) is solar energy. Calling solar energy a pipe dream is naive. Its everywhere. "You are made of star stuff (RIP Carl Sagan)." Calling the corpolitical consumer mindset sustainable... now that is more like a pipe dream.

These more readily renewable sources of energy are what we should have been using for a very long time... Why haven't we been using them in greater amounts? With more efficient machines? Corporations, government, politics. Money really. All sorts of these alternative energy sources and more efficient things have been known for a very very long time... and not implemented for profit reasons. IE Nickel-iron batteries and electric cars we could of had around the time of the model-T... Why weren't they? Because some companies wanted to sell lots of lead-acid batteries with shorter cycle lifes and gasoline because thats where there investments were. There is less profit in a battery you only need to buy once. Large cars and trucks we don't need to buy, but buy anyways because the media tells us to and hey, its cool. We use uranium nuclear reactors because of the plutonium byproduct which the military wants and because the companies were fully invested in uranium at the time... Never mind that thorium is dirt cheap, has by products with shorter half-lifes, and doesn't have a plutonium byproduct as an incentive to stockpile nuclear weapons. And we can't forget that the liquid thorium fluoride reactors are inherently safer than any uranium reactors since they don't depend on computers to shut them off.

Granted there are geothermal, tidal, nuclear, etc... sources of energy which are not necessarily 100% completely from the sun or the gases which coalesced to form our solar system. But in a way they are made possible by the sun none the less...

BTW, next energy era is likely to be the thorium era... then alternative energies or fusion depending on our technological and societal successes or failures.

497 - 7-9-2011 at 00:10

Why not solar thermal? I didn't realize how many options there are for converting high temperature heat and H2O/CO2 into H2/CO + O2.

http://www1.umn.edu/news/features/2011/UR_CONTENT_290042.htm...
http://eands.caltech.edu/articles/LXXII2/CO2_to_Fuel.pdf
http://addis.caltech.edu/publications/Thermochemical%20study...
http://onlinelibrary.wiley.com/doi/10.1002/aic.12580/full
http://en.wikipedia.org/wiki/Cerium%28IV%29_oxide-cerium%28I...
http://www.google.com/patents?id=EbLGAAAAEBAJ&pg=PA14&am...

It seems like there may be plentiful options for the experimenter at home in these directions. Ceria is cheap. Mirrors are not terribly tough. The efficiency doesn't have to be that fantastic to be more attractive than a pricy PV + electrolysis system. Dealing with the high temperature materials on the cheap could be problematic though.
I wish there was more solar thermal chemistry being done in general.

not_important - 7-9-2011 at 05:58

That's a real challenge, getting the very open form of CeO2, then heating to 1500 and quenching to 900 or lower. A bit like making NO by the Birkeland–Eyde process, which also has recently been proposed to be driven by solar heat.


497 - 7-9-2011 at 13:31

That patent forms a porous CeO2 by ball milling the raw powdered oxide with an 70%EtOH + 30%starch mixture, then pressing it into a pellet and sintering at 1500*C. Not exactly "easy" but at least no expensive materials are required.

I didn't get the impression that rapid quenching was required in this situation. Of course the faster it cycles the more productive the process would be.

NO from solar thermal is a great idea..
Venturi quenching with or without cooling fluid injection has been demonstrated to cool the gas quickly enough to catch >50% of the NO.

SO3 seems like another candidate for solar thermal production.

Edit:
Ehh... On further investigation this looks too inconveniently complex for anything but huge scale..
Simplicity and decentralization go well together.
It seems the obvious direction to take that is the microbial route.

Grow a native algae species in locally available wastewater.
Use the same system that delivers CO2 and removes O2 to circulate the algae and foam concentrate some algae up to maybe 3-8%. Anaerobically digest the algae concentrate (at the same temperature as the photobioreactor) and the resulting biogas is bubbled back through the more algae to replace the CO2 fraction with O2 and remove H2S, making it a very lovely fuel... Preferably you could add a carbon rich material like sewage sludge to the digester to improve the C:N and methane yields.

The sludge remaining after digestion is a concentrated nutrient soup that can be used for fertilizer or making more algae. In fact, anaerobic digestion only loses about 16% of the N present. Compare that to around 50% for aerobic composting.

No it's not going to be highly efficient but 6 mil polyethylene is pretty damn cheap.. and there's not much more to it besides a means to pump some air and water. Temperature control would be important. I just read some very interesting things regarding blown in soap foam insulation for greenhouses. Should have very high bang/$. I think it could be applied to other situations quite beautifully. Floating the units in a solar pond might work reasonably well too.

A little less simple, but you could also just (plasma?) reform the desulfurized biogas directly into clean syngas, which could even be scrubbed of remaining CO2 by algae ;D

[Edited on 8-9-2011 by 497]

497 - 4-11-2011 at 04:08

I don't really know where to post this.

I have been pondering a free piston linear alternator steam engine recently, trying to figure out the simplest way to construct one without a machine shop. I had an idea that I'm surprised took me so long to realise. Why bother trying to build a piston cylinder assembly when they're already so plentiful? Why not use a pneumatic cylinder? They may not be designed for steam, but they have basically all the other charictaristics required: pressure ratings up to 250 psi, wear resistant/low friction seals and bearings meant for longterm continuous duty, wide variety of sizes and dimensions, plentiful, and sooo cheap.

I searched around and found no mention of anyone trying to convert them to steam. So I'm wondering why I couldn't build a simple permanent magnet linear alternator, hook it up to a pair of pneumatic cylinders in a push/pull arrangement and run it using a couple solenoid steam valves? The cylinders could be easily ported to run in a uniflow arrangement if needed. The bearings/seals should be durable as there are no side loads at all. If the original seals can't take the heat, repair kits with PTFE seals are cheap. Hell, the whole cylinder is dirt cheap if bought used.

From what I've read, I believe it is possible to construct a linear alternator that is 70-80% efficient without special tools. I don't know how high the total system efficiency could get, but it will surely be similar or better than a traditional small steam engine, and easier to build. The fact that Sandia was able to top 50% with a very similar engine (but using internal combustion) is a good sign. You just can't beat that level of simplicity. If you're not a fan of vibration, you can simple oppose to identical free piston engines.

I realize that using an electric valve system is a limitation on the "rpm" but that could be surmounted by using larger/more cylinders at a slower speed. Or maybe some kind of mechanical or magnetomechanical valve arrangement would be preferable if high power/weight ratio was required? I'm currently contemplating a stationary unit, so cost, simplicity, and efficiency are the priority.

I was originally looking in to stirling engines for this purpose, but they simply cannot achieve that level of design and construction simplicity. And buying a stirling seems to be not an option. The size range I'm interested in is about 1-5 kwh output for now. The heat source will be flue gas from a gasifying biomass burner. The energy will be used to provide space heating, so I thought it was an awful waste to take all that 800*C energy and simply bring it down to 50*C for space heating.

Am I just floating around in dream world here, or is there potential here?

AndersHoveland - 4-11-2011 at 04:22

While I strongly support renewable energy, it is mostly not practical to store electrical energy. When there is no sun or wind, reserve generators (burning methane) will need to be used.

bbartlog - 4-11-2011 at 05:17

Quote: Originally posted by 497  
...Why not use a pneumatic cylinder? They may not be designed for steam, but they have basically all the other charictaristics required: pressure ratings up to 250 psi, wear resistant/low friction seals


I like your thinking. I suspect that the two main issues you would encounter are thermal - the seals are not designed for high temperature working fluid - and overall wear - it's one thing to have a 'continuous duty cycle' where the cylinder operates once every two seconds, but you might be looking at two orders of magnitude more than that.

[Edited on 4-11-2011 by bbartlog]

watson.fawkes - 4-11-2011 at 09:30

Quote: Originally posted by 497  
I have been pondering a free piston linear alternator steam engine recently, trying to figure out the simplest way to construct one without a machine shop. I had an idea that I'm surprised took me so long to realise. Why bother trying to build a piston cylinder assembly when they're already so plentiful? Why not use a pneumatic cylinder?
Perhaps I'm missing something, but isn't what you're talking about tantamount to a Sterling engine?

As for the construction aspects, it's a general rule that if you want to make a new kind of machine, you'll need a machine shop. In this case, you're talking about using small-bore cylinders where you want large-bore ones. It's the line-square relationship between total displacement and sealing surface that matters practically, not the least for reducing friction.

497 - 4-11-2011 at 12:53

Quote:
Perhaps I'm missing something, but isn't what you're talking about tantamount to a Sterling engine?


Not really. As far as I know you can not build a stirling engine with one moving part. Stirling engines also have problems because they have to manage a very high hot side temperature, and high inert gas pressure is required to achieve efficiency.

If you look up schematics of free piston stirling engines, you will find they are not that simple. While possibly easier to construct than a crank shaft style engine, it still looks much more daunting than throwing together a couple pneumatic cylinders. I suppose pneumatic cylinders could be used for constructing a stirling engine too, but the small bore and low pressure rating is more of a limitation on the power output. As far as I know, for a given rpm, displacement, and max pressure a steam engine will be much more powerful than a stirling. A 2' or 3' bore 6' stroke cylinder running at 150-200 psi steam pressure could actually produce useful power right? A similar sized stirling could never yield useful power as far as I understand. Even if multiple cylinders are required, they commonly go for $20-50, so it doesn't really matter.

It wouldn't be much of a "new machine," simple double ended free piston linear alternator engines have been built before, using a variety of pressure sources to operate them.
Some durability can be sacrificed if the engine is cheap/easy enough to construct. Seals are cheap and easy to replace. High temperature fluoropolymer bearings and seals rated to at least 180*C are available.

[Edited on 5-11-2011 by 497]

watson.fawkes - 5-11-2011 at 04:55

I think the largest problem you're going to have it the total cycle rating on the off-the shelf solenoid valves. If they're rated, say, 1,000,000 cycles, and you're running at 100 rpm, then you're only talking about just shy of a week of continuous running. That kind of valve just isn't designed for what you're thinking of doing with it.

bbartlog - 5-11-2011 at 05:42

Quote:
for a given rpm, displacement, and max pressure a steam engine will be much more powerful than a stirling.


Now you've lost me.... don't the parameters you mention pretty well define the power output (I mean I guess you also need min pressure or better the overall shape of the time/pressure in there somewhere but still)? Not saying that a steam engine mightn't be more powerful but if it is it would be due to better numbers in one of those areas...


497 - 5-11-2011 at 09:38

My point was just that a steam engine can developed far higher force on the piston than a stirling. To get much power from a stirling, the working gas must be highly pressurized. Usually to many hundreds of psi. So a pneumatic cylinder is simply not very suitable.

I wondered about solenoid durability. There are so many other other options for valve gear though. I wonder about using a little variable speed electric motor to turn a free spinning ball valve?

Neil - 5-11-2011 at 10:41

For fun, how do you kill a Stirling engine? You lubricate it.

A Stirling piston has to be designed so that it functions with essentially a frictionless seal. Air cylinders and the like have far to much friction to ever be useful in a Stirling engine. if you build a Stirling to run off of huge thermal variations, by the time all is said and done you really just have an inefficient steam engine.

A possibility is to use a closed steam cycle with something like ammonia so that it would function like a steam engine while still having the large energy transfers that gives steam the kick it has.

A Stirling can generate force from very small thermal variations provided the 'working fluid' is large enough. If you put a massive expansion cylinder on a Stirling you can run it off of the heat of your hand.

I have heard that the soviet atomic batteries used Stirling engines tied to generators but I'm not sure if that is true or not.

A purely linear engine is doomed to be inefficient for the same reason a piston engine is doomed, the energy wasted each time the piston is reversed is significant.

Some engines where developed that tied the piston and a pump piston together in a linear format,but they used a rotating flywheel to conserve momentum and drive the piston through the direction shifts, check out Snow engines. In the case of steam, a steam chest with a worn in slide valve is essentially fool proof in operation and nearly indestructible.

The most efficient you can get for recovering steam energy is shoot steam at high pressure through a multistage turbine.


It would be really neat to see a purely linear steam engine in play but in terms of efficiency its not really viable...:(

497 - 5-11-2011 at 21:32

Viable for what? Utility scale? Obviously piston anything will never be. Ever tried to build a 1kw turbine? It doesn't work. Turbines are basically useless for small scale things. Can a small combustion turbine reach 50% efficiency? A linear alternator free piston IC can. Just saying...

Twospoons - 6-11-2011 at 15:33

Quote: Originally posted by 497  


Not really. As far as I know you can not build a stirling engine with one moving part.

Fluidyne. uses liquid "pistons" in vertical tubes.

Quote: Originally posted by 497  

Stirling engines also have problems because they have to manage a very high hot side temperature,


High temp hot side is an essential part of any themal engine (assuming RT as the cold side). Sterling cycle just gets closest to the Carnot limit.

Pneumatic cylinders are not made to handle high temps or steam. The only bits you could keep would be the outer cylinder and piston rod. All the seals would need replacing.

Alternative 'free piston' approach is the pulse tube. Usually mechanically driven to operate as a heat pump, they can also be used as a heat engine. Nice because the only moving parts are the working fluid, and the diaphragm at one end.

497 - 9-11-2011 at 11:01

Ok you got me with the fluidyne. But how practical is that?

I think there is a pretty big difference between one side of the engine being exposed directly to 800 or 1000 degree flue gas versus under 200 for steam less with other fluids...

Seals are pretty cheap. Not hard to replace either.

I guess you could put a pulse tube on either side of a free perminant magnet piston? I fear the pulse tube style operation would perform very poorly at high temeratures. According to wikipedia pulse tube coolers have worse COPs than the alternatives unless the operating temperature is below 80K... I assume the working fluid's density is critical to its efficiency. I wonder if the principle could be somehow adapted to operation with a condensible working fluid? The rediculas level of simplicity is intriguing for sure.

Twospoons - 9-11-2011 at 17:30

The only use I can think of for the fluidyne is pumping water in areas where the only energy source is solar. So its not going to solve the worlds energy crisis. Still pretty neat though - you have to admire its low tech simplicity, and the ability to use low grade heat like solar.

I've often pondered the $ per W for solar thermal electricity vs PV cells, in a small installation (say 2-3kW). Not easy to see where the break point would be.

497 - 12-11-2011 at 12:15

I love the simplicity of the stirling engine, particularly in a free piston arrangement. I think the main issue that needs to be solved is: how can one build a vessel capable of withstanding 10 atm (30 would be better) while being exposed to high temperatures above 600-800*C without expensive equipment and special skills?

That question alone is what makes me lean toward a rankine cycle system, with all its complexity and poor efficiency...

Any suggestions?

Does anyone have any ideas about a simple low tech electrically driven valve system for a rankine cycle piston engine?

celinaaniston86 - 11-1-2012 at 01:04

The wind-power market is intensely backed by loan assures and requires, and like solar power is transforming out to be a wide pit of spent financing that also increases the cost of power to areas whose resources have been necessary to buy its result. It is another ecological pipe dream and one expected to enhance those who go into this doubtful business.

497 - 17-1-2012 at 15:56

The gratzel cell is worth looking up. Cheap to build, decent efficiency.

497 - 19-1-2012 at 21:00

So why the hell hasn't this been brought up before?
http://chemgroups.ucdavis.edu/~mascal/pages/biomass.html
Does it get any easier?? The levulinic acid produced by heating the chloromethylfurfural with water can be converted to high energy density liquid hydrocarbons usable in IC engines by simply pyrolysing the calcium salt at 350-450*. Very high yields overall. Gasification of the remaining lignin can provide excess heat, possibly to generate electricity, etc.
CMF will form the corresponding levulinic ester if heated with an alcohol, so that is another liquid fuel option, although the energy density is a bit lower, 24MJ/kg compared to 34 for the thermally deoxygenated products.
The papers on thermal deoxygenation all talk about using Ca(OH)2 to neutralize the levulinic acid, but I don't understand why straight CaCO3 couldn't be used...

[Edited on 20-1-2012 by 497]

watson.fawkes - 20-1-2012 at 09:28

Quote: Originally posted by 497  
CMF will form the corresponding levulinic ester if heated with an alcohol, so that is another liquid fuel option, although the energy density is a bit lower, 24MJ/kg compared to 34 for the thermally deoxygenated products.
It seems ideal for a small-scale plant. If you first run an ethanol fermentation, the waste from that process should be usable as a feedstock for the CMF one. This could avoid all the energy input problems with pyrolysis.

Just my first thoughts. Don't know about actual feasibility.

GreenD - 20-1-2012 at 09:43

My poiny on solar energy:

You are paying for a renewable source of energy. That is all. If you don't want to pay the extra money because something else is cheaper, the fuck are you? What are your real intentions in life? To save money?

Same shit goes for food. You are paying for either a healthy dose of antioxidants, vitamins and minerals in expensive veggies, and even more expensive in respectable practices of organic food, or you can buy cheap shit and poison yourself and the planet slowly.

Where is the choice in this shit? You're either a fucking idiot who does not think for his or herself and is completely molded by a society driven by the rat race and money, or you understand what & who you are and respect the relationship of your body and the fucking rock it was born on.


My words to anyone who even wants to debate renewable energy (and off topic healthy / organic food):
go die.

PS, I believe UC Davis or Berkeley came up with a >90% photocell. It was about a millimeter^2 in area, and cost was astronomical.

I'd still purchase it.

[Edited on 20-1-2012 by GreenD]

AndersHoveland - 20-1-2012 at 09:52

It should also be remembered that some areas are much more appropriate for solar power than others. Remote desert regions, without less access to electric lines, and with few cloudy days, would be the most appropriate locations for solar power. Ironically, the regions of the world that are the msot enthusiastic about the environment and renewable energy are also the regions of the world with cloudy skies! If the the environmentally-minded residents under cloudy skies want to make the best use of their money, they should pay for the more expensive solar panels in the sunnier regions, to offset the burning of coal by residents who care less about the environment. The country of Norway, in fact, already does this to some extent in African countries.
http://www.flyingblueclubafrica.com/business/business-for-bu...
http://www.sum.uio.no/english/research/news-and-events/event...
Also read about carbon offsets, where you can donate money for solar panels to be used in Africa, instead of installing your own solar panels if you live in a cloudy area. For the same price, you can reduce CO2 emissions more by paying for the solar panels in Africa, where it much sunnier.

[Edited on 20-1-2012 by AndersHoveland]

AndersHoveland - 21-1-2012 at 16:43

One of the potential ways to less expensively store large quantities of electrical energy from solar panels for use on a cloudy day is the vanadium redox battery
http://en.wikipedia.org/wiki/Vanadium_redox_battery

White Yeti - 29-1-2012 at 09:50

I think that more focus should be put into using solar energy to provide heat for industrial processes rather than generating electricity.

If you think about it, we use energy in basically two forms, electricity and heat. We need to figure out how to store heat cheaply and on a massive scale. Molten salts could get the job done, but right now, we use molten salts to boil water and turn turbines to generate electricity instead of heating endothermic industrial processes, like the calcination of calcium carbonate for example.

Solar alone cannot supply all the energy needed for industrial processes, but it can at least help out.

Fifth generation nuclear reactors are working towards just that; why not use solar energy for the same purpose?
According to this source, 70% of the energy industrialized countries use is in the form of heat.

497 - 4-2-2012 at 03:51

https://docs.google.com/viewer?a=v&q=cache:2Z7aIvmX6rkJ:...

This could be a bit a game changer for DIY solar energy. Glycol, PVP, and AgNO3 on glass or polymer results in a random mesh of nanowires that are quite conductive after annealing at 200* while retaining 85% transparency. They performed about as well as current transparent doped oxide coatings overall. Also happens to be flexible to a 4mm radius with no degradation. How cool is that?

Combined with TiO2 particles coated in anthocyanins and saturated with iodide solution, this could be a really simple and low cost solar cell. They call for a Pt layer on the back side of the electrolyte. Not sure if it could be avoided. If not, this looks pretty easy http://pubs.acs.org/doi/abs/10.1021/am900918y

https://docs.google.com/viewer?a=v&q=cache:69lB-W-ga2QJ:...
Another interesting paper, reports that Ag nonowire grids of appropriate dimensions increase efficiency at least 30% above ITO conductive coatings. This is due to the "surface plasmon" effect which promises to greatly increase overall efficiencies. They mention that the Ag nanowires that create the effect can also be used as the transparent electrode. I figure this means we could gain even greater benefit using the solution deposited nanowires which are considerably smaller and more evenly distributed than their grid. 30% efficiency is claimed here http://www.greenoptimistic.com/2008/12/29/surface-plasmons-i... using "metal nanoparticles."

Polymer electrolytes may end up being much easier than the sintered TiO2+liquid method in the Gratzel cell. Need to look into this more. Here's a simple DIY nanostructured ZnO polymer cell http://orbit.dtu.dk/getResource?recordId=227973&objectId... and here is a better polymer and how to make it http://www.sciencemadness.org/talk/files.php?pid=235689&...

This field is so fascinating to me because of the potential great improvements that can be accomplished be simple modifications. The massive implications of said improvements is even more exciting. We could be trying this stuff out now, or waiting 3-5 years for the corporations to sell it to us. Take your pick...

In conclusion, no solar energy is not a pipe dream.

[Edited on 4-2-2012 by 497]

497 - 5-3-2012 at 15:28

I've been saying duckweed is promising for a long time, now here's some proof.
http://m.mailtribune.com/apps/pbcs.dll/article?AID=/20110314...

Same cost as nuclear/coal they say.

How complicated can the system really be?

Why not directly ferment the starch content of the plant (which is at least 5 times more per acre vs corn) then distill off the ethanol to leave the protein and fat intact to be used as high quality feed additive. If you don't need the protein then anaerobic digestion or combustion would yield more energy.

The cost of other waste treatment is greatly reduced by the duckweed, adding further to the benefits.

White Yeti - 5-3-2012 at 16:17

What about using some solar energy to desalinate water? After all, many metals can potentially be mined from the sea, magnesium being one of them. Desalination is the first step. So even though sea salt could be considered a byproduct, it contains all sorts of goodies, like uranium, magnesium, vanadium, manganese and of course, everyone's favourite, GOLD.

Keep in mind that the rest of the world does not have access to safe drinking water. Solving the water shortage first is a better place to start.

Endimion17 - 6-3-2012 at 05:07

Quote: Originally posted by White Yeti  
What about using some solar energy to desalinate water? After all, many metals can potentially be mined from the sea, magnesium being one of them. Desalination is the first step. So even though sea salt could be considered a byproduct, it contains all sorts of goodies, like uranium, magnesium, vanadium, manganese and of course, everyone's favourite, GOLD.

Keep in mind that the rest of the world does not have access to safe drinking water. Solving the water shortage first is a better place to start.


It's already done where it's "economically viable", in some parts of Middle East, though I doubt that the whole life cycle of such process doesn't benefit from all that oil underneath...

Extracting uranium and gold from the sea is highly inefficient. The quantities per unit of volume are too small.

White Yeti - 6-3-2012 at 12:11

Quote: Originally posted by Endimion17  

It's already done where it's "economically viable", in some parts of Middle East, though I doubt that the whole life cycle of such process doesn't benefit from all that oil underneath...

Extracting uranium and gold from the sea is highly inefficient. The quantities per unit of volume are too small.


I agree that the process is not efficient yet, but it's only a matter of time until the technology matures to a point where this will be economically viable. At some point, industry and agriculture will use more water resources than all our freshwater resources combined (think of how much water we need to grow food, process cotton, raise livestock et-cetera).

At some point we will be forced to desalinate water because all our present water resources will either become too polluted to be used or flat out depleted beyond replenishment.

If we are forced to desalinate water to obtain water for agriculture and industry, the salt and all the metals it contains will become a cheap by-product, and someday we will be able to mine valuable metals from it.

Remember what we used to do with crude oil? We used to dig it out of the ground, distil off the kerosene and throw the rest away. There's no reason why extraction technology will not get better as decades roll by. Historically, we've gotten better at separating mixtures into their components, faster, more efficiently and with less resources, there is no reason why we will stop here.

497 - 31-5-2012 at 03:00

http://climate-connections.org/2012/04/02/biomass-insanity-m...

This is just sad... No wonder so many people see biomass as a farce. If you look at the way governments tend to treat it, you will only find more proof that profit is the main driving force here. And sure enough after being repackaged as "green," the taxpayer is getting screwed just as hard. The guys getting paid for their trees don't seem to see the problem with the old way of accounting the carbon... http://nafoalliance.org/featured/forest-owners-again-tell-ep...

http://www.nytimes.com/2010/07/10/business/energy-environmen...

I just hope people won't abandon biomass because of some greedy land/industry owners trying to profit. Algae and duckweed have none of the problems associated with clearing forests, and they grow orders of magnitude faster, while treating waste water and producing starch/protein/oil... There is huge potential here.

A little about duckweed https://docs.google.com/viewer?a=v&q=cache:k1OA-mh4MlEJ:...

10-15tons starch/acre/year = 1200-2000gal ethanol/acre/year
2-3tons high quality protein/acre/year
And algae can easy double/triple those outputs if done right... While corn can get you maybe 2tons starch/acre/year if you use Roundup and a bunch of fertilizer... But which one are we paying for with tax dollars? http://www.showmedaily.org/2010/04/farm-subsidies-are-not-an...

[Edited on 31-5-2012 by 497]

497 - 19-7-2012 at 03:13

Now this is finally starting to look really good!
http://onlinelibrary.wiley.com/doi/10.1002/adma.200904155/ab...
http://onlinelibrary.wiley.com/doi/10.1002/aenm.201100140/ab...
http://www.rsc.org/suppdata/cc/c1/c1cc14687d/c1cc1687d.pdf
http://www.ncbi.nlm.nih.gov/pubmed/21882872
http://www.sciencedirect.com/science/article/pii/S0022024811...

Copper, zinc, tin, sulfur (and sometimes selenium), it couldn't get much cheaper than that...

Oh and it works for high temp efficient thermoelectrics too?
http://repository.upenn.edu/cgi/viewcontent.cgi?article=1185...
http://www.physics.purdue.edu/quantum/files/Yang_CZTS_nl2012...
http://202.127.27.99/selfweb/ketizu/chenlidong/Paper%20PDF/41-ShiXY_XiLL_ChemMater_2010.pdf


White Yeti - 19-7-2012 at 19:37

I listened to a TED talk and I didn't believe what the speaker said; he said that the cumulative capacity of the world's photovoltaic installations is growing exponentially and shows no signs of slowing down.

He was right, take a look:
cumulative-photovoltaic-production-1975-2007-exponential-grwoth-of-solar-power.png - 63kB
The very fact that this kind of growth is taking place shows that solar energy is becoming competitive, or that fossil fuels are becoming antiquated.

For how long this exponential growth will continue remains to be seen, but I truly believe that renewables will not reach their full potential without a smart grid to manage the distribution of power. The next stumbling block will not be power generation, but the management and distribution of power.

497 - 21-7-2012 at 01:55

It took till 2009 for someone to come up with this???
Inflatable parabolic trough collectors... With bonus adjustable focal length.
They say potentially down to 1/20 the cost of traditional parabolics, max 400 C heat output.

http://www.research.fsu.edu/techtransfer/technologyopportuni...

http://www.research.fsu.edu/techtransfer/solarsausage.html

[Edited on 21-7-2012 by 497]

bbartlog - 21-7-2012 at 10:26

Quote:
algae can easy double/triple those outputs if done right


If it were easy, someone would already have done it. Algae is hard. I do like the duckweed, though. That much protein has considerable value, and unlike some of the more efficient algae duckweed is proven to be highly competitive in the wild, i.e. resistant to contamination. And duckweed is far easier to separate.

The inflatable parabolics look like a great low-cost small-scale solution (like for heating house water). I see problems with them scaling to commercial power generation, though: 1/50 weight is not an advantage for wind resistance, and those plastics probably don't have a great MTBF when they get thermally cycled every day.

Dr.Bob - 23-7-2012 at 11:09

Quote: Originally posted by White Yeti  
I listened to a TED talk and I didn't believe what the speaker said; he said that the cumulative capacity of the world's photovoltaic installations is growing exponentially and shows no signs of slowing down.


Yes, Solar is growing fast, but the scale of that slide is still less than 1% of the current energy usage of the world. So even doubling every 2 years, as that graphs shows, it will take about 15 years for Solar to become a major power source, competing with coal, oil, gas and nuclear. I am quite hopeful that solar will achieve breakthroughs in cost and efficiency, that will help, but at current costs, it will be a while before it goes further than 20% of the need, since it can most easily replace peak electric usage, not baseline. And work on energy storage and grid management will help a lot.

So we need more than just solar to really deal with the growing demand for energy, as least for the next 20+ years.

triplepoint - 23-7-2012 at 17:16

I agree with Dr. Bob. Yes, it is true that solar capacity is growing rapidly, but it is starting from such a low baseline that it will not be a major player for quite some time. Load-balancing and the transmission grid are also powerful obstacles to overcome. Also, keep in mind that much of solar's recent growth is due to its being the political flavor of the month, heavily subsidized by governments. Expect the growth rate to slow when the promoters can't deliver on overly optimistic promises. (see solindra bankruptcy, etc.). Look at warren buffet dumping his huge wind power plans when he determined that there was no economically reasonable way to transport the resulting power. There's also no way around the fact that solar is way less energy-dense than the fossil fuels of is touted to replace.